Abstract algebra: Rings and Ideals

Click For Summary
A subset A of a ring S is being analyzed to determine if it qualifies as an ideal, given that S is a Cartesian product of two rings. The discussion highlights that while A has been proven to be a subring of S and one direction of the ideal definition is satisfied, the other direction requires showing that R is commutative. It is noted that R is a ring without zero divisors and without an identity, raising the question of whether such a ring can be commutative. The conclusion drawn is that R cannot be an integral domain since it lacks a unity, indicating that the properties of R do not guarantee commutativity. The exploration emphasizes the complexities involved in proving ideal properties in rings lacking certain characteristics.
lmedin02
Messages
52
Reaction score
0

Homework Statement


The problem is to show that a subset A of a ring S is an ideal where A has certain properties. S is a ring described as a cartisian product of two other rings (i.e., S=(RxZ,+,*)). I have already proved that A is a subring of S and proved one direction of the definition of an ideal. But, the other direction has brought me to having to show that R is commutative. It is given that R is a ring without zero divisors and without identity.


Homework Equations





The Attempt at a Solution


I know that a ring R is commutative if it has the property that ab=ca implies b=c when a is not zero. I have attempted various simple manipulations of this statement by using the fact that R is a ring without zero divisors and without an identity.
 
Physics news on Phys.org
In other words, is a ring R without zero divisors and without an identity commutative.
 
An integral domain has a unity (i.e., identity). In my case, R has no unity so it is not an integral domain.
 
Question: A clock's minute hand has length 4 and its hour hand has length 3. What is the distance between the tips at the moment when it is increasing most rapidly?(Putnam Exam Question) Answer: Making assumption that both the hands moves at constant angular velocities, the answer is ## \sqrt{7} .## But don't you think this assumption is somewhat doubtful and wrong?

Similar threads

  • · Replies 1 ·
Replies
1
Views
2K
  • · Replies 3 ·
Replies
3
Views
1K
  • · Replies 6 ·
Replies
6
Views
1K
Replies
7
Views
3K
  • · Replies 5 ·
Replies
5
Views
934
  • · Replies 3 ·
Replies
3
Views
3K
  • · Replies 4 ·
Replies
4
Views
1K
  • · Replies 3 ·
Replies
3
Views
870
  • · Replies 5 ·
Replies
5
Views
1K
  • · Replies 1 ·
Replies
1
Views
2K